two sides of a triangle are 18 ft and 23 ft. Find the range for the measure of the third side.

Answers

Answer 1

Using the triangle inequality theorem the range of measures of the third side is 5 < x < 41

How to find the third side of the triangle?

The triangle inequality theorem states that any side of a triangle must be shorter than the other two sides added together.

In other words, the triangle inequality theorem describes the relationship between the three sides of a triangle. According to this theorem, for any triangle, the sum of lengths of two sides is always greater than the third side.

Therefore, for a triangle that has the sides a, b and c will follow the principle below:

c < a + b

b < a + c

a < b + c

Therefore, the triangle has two sides as 18 ft and 23 ft. The range of the third side is as follows:

x < 23 + 18; x < 41

18 < 23 + x

23 < x + 18

Therefore, base on the first and third inequality.

x < 41

x > 5

learn more on triangle here: https://brainly.com/question/22554315

#SPJ1


Related Questions

34. G and H are complementary angles.
If the measure of G = (6x-15)° and the measure of H = (3x + 6), find the measure of H.

Answers

Answer: the measure of H is 39 the measure of G is 51

Step-by-step explanation:

6x-15+3x+6=90

9x-9=90

9x=99

x=11

6(11)-15=51

3(11)+6=39

Find the measure of each acute angle in a right triangle in which the measure of one acute angle is 11 times the measure of the other acute angle.

Answers

The measure of the acute angles in the triangle are 7.5 and 82.5

How to determine the measure of the acute angles?

From the question, we have the following parameters:

One acute angle is 11 times the measure of the other acute angle.

Let the acute angles be x and y

Such that

y = 11x

The sum of the acute angles in a right triangle is 90

So, we have

x + y = 90

This gives

x + 11x = 90

So, we have

12x = 90

Divide by 12

x = 7.5

Recall that y = 11x

So, we have

y = 11 x 7.5

Evaluate

y = 82.5

Hence, the acute angles are 7.5 and 82.5

Read more about acute angles at

https://brainly.com/question/17118374

#SPJ1

What is the equation of the line that is perpendicular to y = 2x + 3 and passes through the point (−4, 8). pls show work

Answers

Answer:

y = -1/2x + 6.

Step-by-step explanation:

The line perpendicular to y = 2x + 3 has slope of - 1/2 so we can write it as

y = -1/2x + c

When x = -4, y = 8 so we substitute:

8 = -1/2 (-4) + c

8 = 2 + c

c = 8-2 = 6

So, the answer is

y = -1/2x + 6.

Answer:

y = -1/2x + 6.

Step-by-step explanation:

the temperature has been dropping 2 1/2 degrees every hour and the current tempter is -15 degrees Ferin height how many hours ago was the temperature 0 degrees Ferin height

Answers

The number of hours ago was the temperature 0 degrees is 6 hours.

What is temperature?

Temperature simply means the degree if hotness and coldness in a body.

In this case, the temperature has been dropping 2 1/2 degrees every hour. Let the number of hours be h.

This will be illustrated as:

-15 = -2.5h

where h = number of hours

Divide

h = -15 / -2.5

h = 6

Therefore, the correct option is 6 hours.

Learn more about temperature on:

brainly.com/question/2474268

#SPJ1

A building is in the shape of a square pyramid. each side of the base is 54 meters long and the height is 260 meters. what is the volume of the building? enter your answer in the box. m³

Answers

The volume of the building in the shape of a square pyramid is 252720 m³

In terms of geometry, a square pyramid is a pyramid with a square base and four lateral faces. A pyramid is a type of polyhedron with three or more triangle faces that meet above its base (the apex).

The building is in the shape of a square pyramid.

The base of the building is 54 meters long.

The height of the building is 260 meters.

The volume of the right square pyramid is given as:

V = ( 1/3)a²h

Where a is the base and h is the height.

Now we have,

a = 54 meters and h = 260 meters

Then the volume of the building is:

V = ( 1/3)a²h

V = ( 1/3 ) × ( 54 )² × 260

V = ( 1/3 ) × 54 × 54 × 260

V = 758160/3

V = 252720 m³

Learn more about volume here:

brainly.com/question/12146629

#SPJ4

You start with $100 at a casino and can bet any amount of that money to play a game any amount of times where in each game, you have a 40% chance of winning. If you win, you’ll gain the amount you bet, in addition to being returned the amount you bet. If you lose, you lose your bet. What is your strategy to have the highest chance of getting to $200? (100 - 200 words)

Answers

Our strategy to win $200 is to use $100 in just one chance. It is the best bet for winning $200.

Given that we start with $100 at a casino and can bet any amount of that particular money to play a game to any amount of times where we have a 40% chance of winning in each game. If we win, we will gain the amount we bet and return the amount that we bet. If we lose, we will lose our bet.

It has been asked that what should be our strategy to win get $200.

We can write from the given information that:

Probability of wining = 40 % = 0.4

Probability of losing = 1 - 0.4 = 0.6

The best bet to win $200 is to bet $100 in one go itself.

For the above bet chance of winning $100 is 0.4

But let us assume that we are betting $50 two times,

Then L-Lose and W-Win

There will be 4 rational possible cases (WW,WL,LW,LL)

Probability of all the 4 above given cases will be as follows:

Probability of (WW) = 0.4 X 0.4 = 0.16

Probaility of (WL) = 0.4 X 0.6 = 0.24

Probability of (LW) = 0.6 X 0.4 = 0.24

Probability of (LL) = 0.6 X 0.6 = 0.36

Now for winning $100 we need to win both the bets of $50.

But the probability of winning both the bets = 0.16 ( very less than 0.4)

Hence the best bet would be to use $100 in one chance itself.

To know more about bet click here:

https://brainly.com/question/28813795

#SPJ4

Our strategy to win $200 is to use $100 in just one chance. It is the best bet for winning $200.

It has been asked that what should be our strategy to win get $200.

We can write from the given information that:

Probability of wining = 40 % = 0.4

Probability of losing = 1 - 0.4 = 0.6

The best bet to win $200 is to bet $100 in one go itself.

For the above bet chance of winning $100 is 0.4

But let us assume that we are betting $50 two times,

Then L-Lose and W-Win

There will be 4 rational possible cases (WW,WL,LW,LL)

Probability of all the 4 above given cases will be as follows:

Probability of (WW) = 0.4 X 0.4 = 0.16

Probability of (WL) = 0.4 X 0.6 = 0.24

Probability of (LW) = 0.6 X 0.4 = 0.24

Probability of (LL) = 0.6 X 0.6 = 0.36

Now for winning $100 we need to win both the bets of $50.

But the probability of winning both the bets = 0.16 ( very less than 0.4)

Hence the best bet would be to use $100 in one chance itself.

learn more about of probability here:

https://brainly.com/question/11234923

#SPJ4

nathan is making a beaded bracelet. for every 5 red beads there are 3 white beads. what is the ratio of red to white beads?

Answers

Answer:

5;3

Step-by-step explanation:

Kelsey has an orange ribbon and a blue ribbon. The orange ribbon is 7 11/12 inches long and the blue ribbon is 17 5/12 inches long. How much longer is the blue ribbon than the orange ribbon?

Answers

The blue ribbon is 2.2 times longer than the orange ribbon

How to determine the ratio

From the information given, we have that;

The orange ribbon is 7 11/12 inches long The blue ribbon is 17 5/12 inches long

Now, lets convert the mixed fractions to improper fractions;

Length of orange ribbon = 95/12

Length of blue ribbon = 209/12

The ratio is expressed as;

Length of blue ribbon/length of orange ribbon

Substitute the values, we have;

= 209/12 ÷ 95/12

Take the inverse of the divisor and multiply

= 209/ 12 × 12/95

= 2. 2 times

Hence, it is 2.2 times longer.

Learn more about ratio here:

https://brainly.com/question/2784798

#SPJ1

Help please its due tonight

Answers

Answer:

[tex]\sf 12-5x+6x=4[/tex]  (Given)

[tex]\sf 12+x=4[/tex]  (Combine like terms)

[tex]\sf x=-8[/tex]  (Subtraction property of equality)

--------------------------------------------------

Hope this is what you're looking for!

Answer:

1. given

2. combine like terms,

3. subtraction property of equality

Step-by-step explanation:

Tell whether it shows growth or decay and graph by using a table of values

Answers

SOLUTION

Given the question in the image, the following are the solution steps to answer the question.

STEP 1: Write the given exponential function

[tex]f(x)=0.5^x[/tex]

STEP 2: Compare with the standard exponential function to determine whether it shows growth or decay.

If a is positive and b is greater than 1 , then it is exponential growth.

If a is positive and b is less than 1 but greater than 0 , then it is exponential decay.

[tex]\begin{gathered} f(x)=a\cdot b^x_{} \\ \text{Comparing with }f(x)=0.5^x,\text{ it can be written as }f(x)=1\times0.5^x, \\ It\text{ can be se}en\text{ that }a=1,b=0.5 \\ \text{ Since }a\text{ is positive and b is less than 1 but greater than 0, then it is an exponential decay.} \end{gathered}[/tex]

Hence, the given function shows an exponential decay.

STEP 3: Graph the values by using a table of values

We generate a table of values first

STEP 4: Plot the graph of the data values

The graph shows y varying directly as x according to the equation y = kx. What is the value of d?
A d = kb + kdd = kb + kd
B d = ka + kcd = ka + kc
C d = b + kdd = b + kd
D d = kb

Answers

The equation that represents the value of d is d = b + kc

How to determine the value of d?

The graph that completes the question is added as an attachment

The equation of the graph is given as

y = kx

Because the graph shows a direct variation, the equation can be illustrated as

y = mx

Where m represents the slope of the line

By comparing the equation y = mx and y = kx, we can conclude that

m = k

This means that the slope of the equation y = kx is k

From the graph, we have the points

(x, y) = (a, b) and (a + c, d)

The slope of a line is calculated using

Slope = (y₂ - y₁)/(x₂ - x₁)

Where

Slope = k

(x₁, y₁) = (a, b) and (x₂, y₂) = (a + c, d)

So, we have

k = (d - b)/(a + c - a)

Evaluate the like terms

k = (d - b)/(c)

Multiply both sides by c

d - b = kc

Add b to both sides

d = b + kc

Hence, the value of d is d = b + kc

Read more about linear equations at

https://brainly.com/question/4074386

#SPJ1

Juanita wants to buy a computer. She needs to earn at least 400 dollars. If she makes 9 dollarsan hour and has already saved 40 dollars, how many hours does she need to work? Write an inequality and solve.

Answers

Problem

Juanita wants to buy a computer. She needs to earn at least 400 dollars. If she makes 9 dollars an hour and has already saved 40 dollars, how many hours does she need to work?

Solution

Let x the number of hours required we can set up the following inequality:

40 + 9x > 400

And now we can solve for x we can subtract 40 and we got:

9x > 360

And then we got:

x> 40

So then Juanita needs to work at least 40 hours

And the graph of the inequality is given by:

⦁ At store A, 3 pounds of apples cost $12. ⦁ At store B, the cost is given by y = 2x where y is the cost in dollars and x is the number of pounds of apples. ⦁ The cost of apples at store C is shown in the graph. Answer the questions to find out which store's apples are the least expensive. 1. What is the unit price of apples at store A? Explain how you found this unit price. (2 points) 2. What is the unit price of apples at store B? How did you use the equation to find the unit price? (2 points) 3. What is the unit price of apples at store C? How did you use the graph to find the unit price? (3 points) 4. Which store has the lowest unit price for apples? (3 points)

Answers

1. The unit price of apple in store A is $4

2. The unit price of apple in store B is $2

3. The unit price of apple in store C is $3

4. The store with the lowest unit price is store B

1. How to determine the unit price at store A

We can obtain the unit price at store A by doing the following:

3 pounds of apples cost $12.

Therefore,

1 pound will cost = (1 × pound $12) / 3 pounds = $4

Thus, the unit price is $4

2. How to determine the unit price at store B

We can obtain the unit price at store B by doing the following:

y = 2x

Number of pound (x) = 1Cost (y) = ?

y = 2x

y = 2 × 1

y = $2

Thus, the unit price is $2

3. How to determine the unit price at store C

We can obtain the unit price at store C by obtaining the slope of the graph. This can be obtaioned as follow:

x coordinate 1 (x₁) = 1x coordinate 2 (x₂) = 3y coordinate 1 (y₁) = 3y coordinate 2 (y₂) = 9Slope (m) =?

m = (y₂ – y₁) / (x₂ – x₁)

m = (9 – 3) / (3 – 1)

m = 6 / 2

m = 3

Thus, the unit price is $3

4.How todetermine the store with the lowest unit price

Unit price at store A = $4Unit price at store B = $2Unit price at store A = $3

From the above, we can conclude that store B has the lowest unit price

Learn more about unit price:

https://brainly.com/question/12611440

#SPJ1

a professor wants to study the effectiveness of a new study tool for a course. there are 150150150 students registered for the course. the professor ranks the students according to cumulative average in the prerequisite course. for every 222 students, in order, from the list, the professor flips a coin to assign one student to use the new study tool and the other to use the previous study tool. what type of experiment design is this?

Answers

It is a completely randomized design.

What is completely randomized design ?

Completely randomized designs are used in experiment design to explore the effects of a single primary factor without taking additional nuisance variables into account. This article describes single-factor, entirely randomized designs. On the basis of the various levels of that primary factor, the experiment compares the values of a response variable. The levels of the main factor are given at random to the experimental units in fully randomized designs.

Objects or subjects are assigned to groups in a completely random design at random. Labeling each subject before choosing from the labeled participants in a table of random numbers is one common practice for allocating subjects to treatment groups. Computers can be used to accomplish this as well.

To learn more about completely randomized design click here:

https://brainly.com/question/16024549

#SPJ4

please help like soon

Answers

The distance between the two points is 10 units and the slope of the line is -78/25

Distance Between Two Points

To find the distance between two points, we can simply use the formula of distance between two which is given as

[tex]d=\sqrt{(y_2-y_1)^2 + (x_2 - x_1)^2}[/tex]

Let's substitute the values and solve for the distance.

[tex]d=\sqrt{(y_2-y_1)^2 + (x_2 - x_1)^2}\\d = \sqrt{(1-7)^2 + (-3 - 5)^2} \\d = 10[/tex]

2)

The slope of the line can be found using the two points given.

[tex]m = \frac{y_2 - y_1}{x_2 - x_1} \\[/tex]

let's substitute the values and solve.

[tex]m = \frac{y_2 - y_1}{x_2 - x_1} \\\\m = -\frac{78}{25}[/tex]

Learn more distance between two points here;

https://brainly.com/question/7243416

#SPJ1

a sunflower is planted in a garden and the height of the sunflower increases by 10% per day. 2.66 days after being planted the sunflower is 10.3 inches tall. what is the 1-day growth factor for the height of the sunflower? 0.10 incorrect how tall is the sunflower 3.66 days after being planted? incorrect inches how tall is the sunflower 4.66 days after being planted? incorrect inches what is the 2-day growth factor for the height of the sunflower?

Answers

Two day growth factor for the height of the sun flower is 1.1449

Take the current number and subtract it from the prior value to determine the growth rate. The growth rate is then expressed as a percentage by multiplying the difference by the previous number and dividing by 100.

A height increases by 7% per day

So the growth factor for height of sun flower is  107/100 =1.07

For 2.7 days - 12 inch

3.74 days increase by 7% by day

⇒ 12 + 7/100 x 12

⇒ 12 ( 107/100 )

⇒ 12.84 inches

3.74 days 12.84 inches

4.74 days increase by 7 % by day

⇒ 12.84 + 7/100 x 12.84

⇒ 12.84 x 107/100

⇒ 13.73 inches

Two days growth factor

⇒ 107/100 x 100/107

⇒ 1.1449

Hence Two day growth factor for the height of the sun flower is 1.1449

To know more about growth factor visit :

https://brainly.com/question/12052909

#SPJ4


XYZ and LMN are complementary angles. XYZ = (3x+14)° and LMN= (5x+24)°. What is LMN in degrees?
Write your answer as a decimal.

Answers

Answer:

6.5°

Step-by-step explanation:

We know that both XYZ and LMN are complementary angles.

Complementary angles add up to 90°.

So,

we can simply add both of the expressions and set it equal to 90 (degrees).

(3x+14) + (5x+24) = 90

-> add the variables and constants

8x+38=90

-> subtract the 38 from both sides (90-38)

8x=52

-> divide out the coefficient to isolate the variable

x=6.5

So, the answer is 6.5 degrees.

Hope this helped :)

A new club sent out 152 coupons to boost sales for next year's memberships. They provided 3 times as many to potential members than to existing members. How many coupons did they send to existing members?

Answers

The club has to send 57 coupons to existing members.

Given,

The number of coupons sent out by a new club to boost sales for next year's membership = 152

They provide 3 times as many to potential members than to existing members

We have to find the number of coupons they send to existing members;

Here,

x be the number of coupons to existing members.

Then,

3x = 152

Now,

x = 152/3 = 50.66 ≈ 57

That is,

The club has to send 57 coupons to existing members.

Learn more about number of coupons here;

https://brainly.com/question/28736307

#SPJ1

Jason bought four tickets to a basket ball game. He paid a total of $42.10 for the tickets and his parking was $6.50. What is the cost of each ticket

Answers

The cost of each ticket is 8.9 dollars.

What is cost price?

In mathematics, the term cost price can be explained with the help of profit which states, as the amount which is on the commodity and in the form of standard price called cost price. For the profit, the amount of the commodity can be designed by the shopkeeper which is bigger amount as per the cost price.

According to the question, the Jason bought 4 tickets for basket ball game. He paid a total cost of $42.10 for the tickets and for parking as $6.50

The paid amount for the 4 tickets is: $42.10.

And the parking cost is: $6.50

Total cost price without parking is: $42.10 - $6.50 = $35.60

Therefore, to calculate the cost of each ticket is: 35.60/4 = 8.9 dollars

Hence, the cost of each ticket is 8.9 dollars.

To learn more about the cost price from the given link:

brainly.com/question/1078746

#SPJ9

A local jewelry store sells class rings, which it purchases for $60 each. The store engraves a name and date on the ring and sells it using a markup rate of 340%.

Answers

Class rings are sold by a nearby jewelry store, which buys them for $60 each. The amount of markup is y=204.

Given that,

Class rings are sold by a nearby jewelry store, which buys them for $60 each. The ring has a name and date engraved on it, and the retailer markup it up by 340% before selling it.

We have to find the equation that can be used to find the amount of markup y.

So,

We can write the equation as

Amount of markup=340% of the purchases amount

Taking amount of the markup as y and purchases amount is $60.

y=340%×60

y=340/100×(60)

y=34×6

y=204

Therefore, The amount of markup is y=204.

To learn more about amount visit: https://brainly.com/question/13024617

#SPJ9

5/6a-3=5/8

How would you solve this?

Answers

Step-by-step explanation:

[tex] \frac{5}{6} (a) - 3 = \frac{5}{8} \\ lcd = 48 \\ \\ \frac{5}{6} (a)(48) - 3(48) = \frac{5}{8} (48) \\ 40a - 144 = 30 \\ 40a = 30 + 144 \\ 40a = 174 \\ \frac{40a}{40} = \frac{174}{40} \\ a = 4\frac{14}{40} \\ a = 4 \frac{7}{20} [/tex]

I FIRST FOUND THE LCD AFTER THAT MULTIPLY EACH AND EVERY TERM IN THE EQUATION BY IT TO HET RID OF THE FRACTIONS AND SOLVE FOR a.

A cookie factory uses 1/3 of a barrel of oatmeal in each batch of cookies. The factory used 2/3 of a barrel of oatmeal yesterday. How many batches of cookies did the factory make?

Answers

Answer:

it would be 89

Step-by-step explanation:

Jocelyn has a points card for a movie theater.
. She receives 25 rewards points just for signing up.
• She earns 12.5 points for each visit to the movie theater.
. She needs at least 170 points for a free movie ticket.
Write and solve an inequality which can be used to determine v, the number of visits
Jocelyn can make to earn her first free movie ticket. Inequality?

Answers

Answer: 25 + 12.5v ≥ 170

Step-by-step explanation:

We already have the 25 reward points; it also gives the information that she gets 12.5 points for each visit. Because we don't know the amount she has gone to the theater or will go we have to use a variable; 12.5v. When we add them, we get 25 + 12.5v. The inequality we use is ≥ because it says "at least". When this is used that means it has to be equal to or greater and that is why use that sign and get the full equation;

5 + 12.5v ≥ 170

Answer:

55

Step-by-step explanation:

Evaluate the following expressions picture attached 100 POINTS

Answers

Answer:

3 5/14 (second choice)

Step-by-step explanation:

c = -3 1/7

d = 6.5 (6 1/2 as fraction)

-3 1/7 as improper fraction: -22/7

6 1/2 as improper fraction: 13/2

Now put both fractions into a common denominator

-22/7 = -44/14

13/2 = 91/14

Now add:

-44/14 + 91/14 = 47/14

Change 47/14 back to fraction

47/14 = 3 5/14

You buy a container of cat litter for $13.75 and a bag of cat food for x dollars. The total purchase is $20.80, which includes 4% sales tax. Write and solve an equation to find the cost of the cat food.

Answers

The cost of the bag of cat food is $6.22

What is cost prize ?

The prize at which the good and services have been bought is known as cost prize.

here, the given information is :

Cost of a container of cat liter = $13.75

Cost of a bag of cat food = X dollars

Total amount of money required to purchase the items = $20.80

Percentage of sales tax that had to be given = 4%

Then,

Amount of sales tax given = (4/100) * 20.8 dollars

                                            = 20.8/25 dollars

                                            = 0.83 dollars

Then the total purchase price for the two items = 20.80 - 0.83 dollars

                                                                               = 19.97 dollars

Now we can easily find the cost of the cat food by subtracting the cost of the liter from the total purchase amount :

Cost of the cat food = 19.97 - 13.75 dollars

                                = 6.22 dollars

Therefore, the cost of the bag of cat food is $6.22

check and know more about cost price and discount here :

https://brainly.com/question/11027396

#SPJ1

HELP PLS I NEED THIS ANSWERED QUICKLY
A membership at Gym A costs $50 for 5 months. A membership at Gym B down the street costs $40 for 3 months. You write two equations in the form of y=kx to try and figure out which membership would be cheaper for a year. What is the value of k for the cheaper membership?

Answers

The cost for a year is cheaper in the Gym A than Gym B. The value of k for the cheaper membership would be; $10.

What is an equation?

An equation is an expression that shows the relationship between two or more numbers and variables. A linear equation is an equation that increases at a constant rate.

An equation in the form y = kx is called a linear equation.

In the equation of form y = kx

y = dependent variable which is the total cost

x = number of months

k = constant of proportion / cost per month

Thus k = y/x

The Cost per month at Gym A = $50 / 5 = $10

The Cost per month at Gym B = $40 / 3 = $13.33

The Membership cost at Gym A for a year = $10 x 12 = $120

The Membership cost at Gym B for a year = $13.33 x 12 = $160

Hence, The cost for a year is cheaper in the Gym A than Gym B. The value of k for the cheaper membership would be; $10.

To learn more about linear functions, please check: brainly.com/question/26434260

#SPJ1

The diagram shows the positions of three turbines A, B and C.
A is 6 km due north of turbine B.
C is 4.5 km due west of turbine B.
(a) Calculate the distance AC.

Answers

the distance AC is 7.5cm

Calculator
What is the area of a rectangle with vertices at
ter your answer in the box.
(0,-4).(-1, -3) (2, 0), and (3, − 1)?

Answers

Answer:

The answer to your question is: 6 units

Step-by-step explanation:

Data

A (-4, 0)

B (-3,1)

C (0,-2)

D (-1,-3)

Formula

d = √((x2 - x1)² + (y2 - y1)²)

Process

dAB = √((x2 - x1)² + (y2 - y1)²)                 dAC = √((0 + 4)² + (-2 + 0)²)

      = √((-3 + 4)² + (1 - 0)²)                      dAC = √((4)² + (-2)²)

      = √((1)² + (1)²)                                    dAC = √(16 + 4)

      = √1 + 1                                             dAC = √20

dAB = √2  = dCD

dAD = √((-1 + 4)² + (-3 - 0)²)                    

dAD = √((3)² + (-3)²)

dAD = √9 + 9

dAD = √18 = dBC

reactangle's area = base x height = √18 x √2 = √18 x 2 = √36

                                                       = 6 units

The answer is six units

Measure of angle 1 = 110 degrees. You only need to type 110.

Answers

The unknown angles are as follows:

∠1 = 110°∠2 = 70°∠3 = 110°∠4 = 70°∠5 = 110°∠6 = 70°∠7 = 110°∠8 = 70°How to find angles?

When parallel lines are cut by a transversal line, angle relationships are formed such as corresponding angles, linear angles, alternate angles etc.

Therefore,

∠1 = 110 degrees

∠2 = 180 - 110 (sum of angles on a straight line)

∠2 = 70°

∠3 = ∠1 (vertically opposite angles)

Vertically opposite angles are congruent. vertically opposite angles share the same vertex

Therefore,

∠3 = 110°

∠2 = ∠4 (vertically opposite angles)

∠4 = 70°

∠1 = ∠5 (corresponding angles)

Corresponding angles are congruent

∠5 = 110°

∠6 = ∠4(alternate angles)

Alternate angles are congruent.

∠6 = 70°

∠1 = ∠7 (alternate exterior angles)

∠7 = 110°

∠2 = ∠8(alternate exterior angles)

∠8 = 70°

learn more on angles here: brainly.com/question/18166236

#SPJ1

to collect these data, the researchers randomly selected american households to visit and interviewed the adult member of the household whose birthday was nearest. is this an experiment or an observational study?

Answers

In an observational study, we measure or survey members of a sample without trying to affect them. In an experiment, we assign people or things to groups and apply some treatment to one of the groups, while the other group does not receive the treatment.

This is clearly an observational study

Learn more about observational study:

brainly.com/question/17269327

#SPJ4

Other Questions
HEPP!!Which system of checks and balances checks this and why?The president vetoes a bill that has strong support in the House of Representatives and the Senate Rewrite the simple past sentences from the active voice into the passive voice.1.Alexander Flemming didnt discover penicillin in 1915.2.Two million people did not use the London Underground system last year.3.They didnt teach Chinese in our school.4.They didnt offer us anything to drink.5.Someone did not turn the lights off.help me pls!! what is the molality of a solution contain 82.6g of butane dissolved in 0.500kg of hexane why was it important that new england and chesapeake were different Compare the monthly payments and total loan costs for the following pairs of loan options. Assume that both loans are fixed rate and have the same closing costs.You need a $140,000 loan.Option 1: a 30-year loan at an APR of 9%.Option 2: a 15-year loan at an APR of 8.5% A molecule contains 3 atoms. It has 1 triple bond and 1 single bond. There are no lone pairs on any of the atoms. How many rheds are within the molecule?. Which is true of the force pair of Newton's third law?a.)The two forces are in the same direction.b.)The two forces never produce an acceleration. c.)The two forces act on different objects. The Sanchez family had dinner at their favorite restaurant. A 9% salestax was added to their bill. Amy paid the bill with a $10 gift certificateplus $30.60. How much did the family's dinner cost before tax? Roundyour answer to the nearest penny.Hint: Use the equation P+rP=T,where P is the price before tax, r is the sales tax rate, and T is the totalprice, including tax. buyers believe they have been victims of discrimination from a real estate broker. the buyers filed a complaint, and the next step is that a) hud will begin an investigation. b) hud will take the broker's license until the case is settled. c) the broker is considered to be guilty and will be watched until the investigation is complete. d) the buyers must prove that the discrimination occurred to continue the investigation. what expedition did thomas jefferson establish to explore the west under the leadership of lewis and clark? What are the degree and leading coefficient of the polynomial?Y^3+8y+12y^5-6 which situation is best modeled by an exponential distribution? tafadzwa runs between 10 and 16 laps every day. his times get slower as he gets tired, but his daily average is 48.2 s per lap. what is the probability that his mean lap time tomorrow will be less than 45 s? suppose that meteors large enough to cause mass extinction strike the earth at regular, although exceedingly large, intervals. what is the probability that a large meteor strike will cause a mass extinction event within the next 1000 years? on a dark desert highway, drivers fall asleep and run off the road randomly. what is the probability that a driver will run off the road in one particular 10-mile stretch of this highway? suppose that people call in sick randomly and independently of one another at a business called acme anvil supply. on average, one person calls in sick per week. what is the probability that no one will call in sick next week? suppose that 0.76% of tires are defective, and defective tires occur independently of one another. what is the probability that no more than three of the next 100 tires are defective? Drag the events shown in chronological order, with the first event to occur at the top and the last event at the bottom.President Lincoln is assassinatedthe union loses Fort SumterGeneral Sherman wages total war across Georgia in the march to the Sea campaignConfederate General Lee retreats at the Antietam, allowing the Union to claim victory General Lee surrenders at AppomattoxLincoln issues the Emancipation Proclamation Examine the electron configurations of oxygen, phosphorus and gallium. According to valence bond theory, how many bonds could each of these atoms form?. Solve the given system of equations.S =t =-6s+ 3t = - 45-s-5t= -2 Find the volume of this cone. Use 3 for TT. V = 7 r26 3 8ft V [?]ft V 1 6ft Enter SRI nhte. Das under sfac no. 8, conceptual framework for financial reporting, chapter 4, elements of financial statements, interrelated elements of financial statements include distributions to owners notes to financial statements distributions to owners yes notes to financial statements no distributions to owners no notes to financial statements yes distributions to owners yes notes to financial statements yes distributions to owners no notes to financial statements no 76,300,000 in scientific notation What are some different sentences a criminal might face? Que propose le Bret a Cyrano aux vers14-15? Cyrano accepte t-il cette propositions ?quelle raison invoque t-ilCYRANO, avec un rire amerQue jaimasse ?(Changement de ton et gravement.)Jaime.LE BRETEt peut-on savoir ? Tu ne ma jamais dit ?CYRANOQui jaime ? Rflchis, voyons. Il minterditLe rve dtre aim mme par une laide,Ce nez qui dun quart dheure en tous lieux me prcde ;Alors moi, jaime qui ? Mais cela va de soi !Jaime mais cest forc ! la plus belle qui soit !LE BRETLa plus belle ?CYRANOTout simplement, qui soit au monde !La plus brillante, la plus fine,(Avec accablement)La plus blonde !LE BRETEh, mon Dieu, quelle est donc cette femme ?CYRANOUn dangerMortel sans le vouloir, exquis sans y songer,Un pige de nature, une rose muscadeDans laquelle lamour se tient en embuscade !Qui connat son sourire a connu le parfait.Elle fait de la grce avec rien, elle faitTenir tout le divin dans un geste quelconque,Et tu ne saurais pas, Vnus*, monter en conque*,Ni toi, Diane*, marcher dans les grands bois fleuris,Comme elle monte en chaise et marche dans Paris !LE BRETSapristi ! Je comprends. Cest clair !CYRANOCest diaphane.LE BRETMadeleine Robin, ta cousine !CYRANOOui, Roxane.LE BRETEh bien ! mais cest au mieux ! Tu laimes ? Dis-le-lui !Tu tes couvert de gloire ses yeux aujourdhui !CYRANORegarde-moi, mon cher, et dis quelle esprancePourrait bien me laisser cette protubrance* !Oh ! je ne me fais pas dillusions ! Parbleu,Oui, quelquefois, je mattendris, dans le soir bleu ;Jentre en quelque jardin o lheure se parfume ;Avec mon pauvre grand diable de nez je hume*Lavril, je suis des yeux, sous un rayon dargent,Au bras dun cavalier, quelque femme, en songeantQue pour marcher, petits pas, dans de la lune,Aussi moi jaimerais au bras en avoir une,Je mexalte, joublie et japerois soudainLombre de mon profil sur le mur du jardin !LE BRET, muMon ami !CYRANOMon ami, jai de mauvaises heures !De me sentir si laid, parfois, tout seulLE BRET, vivement, lui prenant la mainTu pleures ?CYRANOAh ! non, cela, jamais ! Non, ce serait trop laid,Si le long de ce nez une larme coulait !